LSAT 48 – Section 4 – Question 15

You need a full course to see this video. Enroll now and get started in less than a minute.

Target time: 1:11

This is question data from the 7Sage LSAT Scorer. You can score your LSATs, track your results, and analyze your performance with pretty charts and vital statistics - all with a Free Account ← sign up in less than 10 seconds

Question
QuickView
Type Tags Answer
Choices
Curve Question
Difficulty
Psg/Game/S
Difficulty
Explanation
PT48 S4 Q15
+LR
Flaw or descriptive weakening +Flaw
A
14%
159
B
0%
158
C
2%
157
D
79%
164
E
4%
158
137
148
159
+Medium 146.544 +SubsectionMedium

This is a pretty tough question. We're prone to understand the argument incorrectly.

The conclusion states that the museum's continued existence depended on the coverage from the local media. In other words, the local media's coverage was a necessary condition for the museum's being still open and in business.

Fair enough. Why should we believe this? One premise says that if there was low attendance at the recent exhibit then the museum would have closed. Okay, good. This means that a necessary condition of the museum staying open is medium-to-high level attendance at the recent exhibit.

Now, in order for the conclusion to follow, we simply need to show that coverage from the local media was the only thing responsible for medium-to-high level attendance.

Does the remaining premises show that? No. It says that local media coverage existed. It also says that local media coverage seems to have contributed to attendance. In other words, it seems to have been a causal factor.

Seems to have contributed? Did it contribute or not? Was it a causal factor or not? We don't know.

That's mistake #1.

Correcting for it, the premise still isn't good enough. The corrected version says that local media coverage contributed to attendance. We we needed it to say that local media coverage was the necessary cause for medium-to-high level attendance. Otherwise, why would the museum's existence depend on it? That's mistake #2.

As for (A), the "necessary condition" it's referring to is "medium-to-high level attendance at the recent exhibit." But the argument never mistook that for a sufficient condition. Did you?

Take PrepTest

Review Results

Leave a Reply